Câu 3.3 trang 86 sách bài tập đại số và giải tích 11 nâng cao

\({1 \over 2}.{3 \over 4}.{5 \over 6}...{{2k + 1} \over {2k + 2}}.{{2k + 3} \over {2k + 4}} < {1 \over {\sqrt {3n + 4} }}.{{2k + 3} \over {2k + 4}}\,\,\,\,\,\,\,\,\,\,\,\,\,\,\,\,\,\,\,\,\,\,(3)\)
Lựa chọn câu để xem lời giải nhanh hơn
  • LG a
  • LG b

Chứng minh rằng với mọi số nguyên dương n, ta luôn có các bất đẳng thức sau:

LG a

\({1 \over {n + 1}} + {1 \over {n + 2}} + ... + {1 \over {3n + 1}} > 1\)

Lời giải chi tiết:

Ta sẽ chứng minh

\({1 \over {n + 1}} + {1 \over {n + 2}} + ... + {1 \over {3n + 1}} > 1\) (1)

Với mọi \(n \in N^*,\) bằng phương pháp quy nạp.

Với \(n = 1,\) ta có

\({1 \over 2} + {1 \over 3} + {1 \over 4} = {{13} \over {12}} > 1.\)

Như vậy, (1) đúng khi \(n = 1.\)

Giả sử đã có (1) đúng khi \(n = k,k \in {N^ * }\), tức là

\({1 \over {k + 1}} + {1 \over {k + 2}} + ... + {1 \over {3k + 1}} > 1,\)

Ta chứng minh (1) cũng đúng khi \(n = k + 1,\) nghĩa là ta sẽ chứng minh

\({1 \over {k + 1}} + {1 \over {k + 2}} + ... + {1 \over {3k + 1}} + {1 \over {3k + 2}} + {1 \over {3k + 3}} + {1 \over {3k + 4}} > 1\)

Thật vậy, ta có

\({1 \over {k + 1}} + {1 \over {k + 2}} + ... + {1 \over {3k + 1}} + {1 \over {3k + 2}} + {1 \over {3k + 3}} + {1 \over {3k + 4}}\)

\( = {1 \over {k + 1}} + {1 \over {k + 2}} + ... + {1 \over {3k + 1}} + {1 \over {3k + 2}} + {1 \over {3k + 3}} + {1 \over {3k + 4}}\)

\(- {1 \over {k + 1}}\)

\( = {1 \over {k + 1}} + {1 \over {k + 2}} + ... + {1 \over {3k + 1}} + {2 \over {3(k + 1)(3k + 2)(3k + 4)}}\)

\( > {1 \over {k + 1}} + {1 \over {k + 2}} + ... + {1 \over {3k + 1}} > 1\) (theo giả thiết quy nạp).

Từ các chứng trên suy ra (1) đúng với mọi \(n \in N^*\)

LG b

\({1 \over 2}.{3 \over 4}.{5 \over 6}...{{2n + 1} \over {2n + 2}} < {1 \over {\sqrt {3n + 4} }}\)

Lời giải chi tiết:

Ta sẽ chứng minh

\({1 \over 2}.{3 \over 4}.{5 \over 6}...{{2n + 1} \over {2n + 2}} < {1 \over {\sqrt {3n + 4} }}\)

Với mọi \(n \in N^*,\) bằng phương pháp quy nạp.

Với \(n = 1,\) ta có

\({1 \over 2}.{3 \over 4} = {3 \over 8} < {1 \over {\sqrt {3.1 + 4} }}\) ( vì \(9.7 = 63 < 64 = {8^2}\) ).

Như vậy, (2) đúng khi \(n = 1.\)

Giả sử có (2) đúng khi \(n = k,k \in {N^ * }\). Khi đó, ta có

\({1 \over 2}.{3 \over 4}.{5 \over 6}...{{2k + 1} \over {2k + 2}}.{{2k + 3} \over {2k + 4}} < {1 \over {\sqrt {3n + 4} }}.{{2k + 3} \over {2k + 4}}\,\,\,\,\,\,\,\,\,\,\,\,\,\,\,\,\,\,\,\,\,\,(3)\)

Lại có : \({(2k + 3)^2}.(3k + 7) < {(2k + 3)^2}.(3k + 7) + k + 1\)

\(= (3k + 4){(2k + 4)^2}.\)

Do đó : \({1 \over {\sqrt {3n + 4} }}.{{2k + 3} \over {2k + 4}} < {1 \over {\sqrt {3n + 7} }}.\,\,\,\,\,\,\,\,\,\,\,\,\,\,\,\,\,\,\,\,\,\,\,\,\,\,\,\,\,\,\,\,\,\,\,\,\,\,\,(4)\)

Từ (3) và (4) suy ra

\({1 \over 2}.{3 \over 4}.{5 \over 6}...{{2k + 1} \over {2k + 2}}.{{2k + 3} \over {2k + 4}} < {1 \over {\sqrt {3k + 7} }},\)

Nghĩa là ta cũng có (2) đúng khi \(n = k + 1.\)

Từ các chứng minh trên suy ra (2) đúng với mọi \(n \in N^*\).